LSAT and Law School Admissions Forum

Get expert LSAT preparation and law school admissions advice from PowerScore Test Preparation.

User avatar
 Dave Killoran
PowerScore Staff
  • PowerScore Staff
  • Posts: 5972
  • Joined: Mar 25, 2011
|
#41184
Complete Question Explanation
(The complete setup for this game can be found here: lsat/viewtopic.php?t=15652)

The correct answer choice is (A)

The question stems in these two questions each establish that O must live east of the MK block. For this to occur, O must live in houses 5, 6, or 7. Consequently, O, P, and L must occupy the fifth, sixth, and seventh houses in some order, and M must live in the third house:

Dec91_Game_#2_#12_diagram 1.png
Only N and R remain unplaced, and we can infer from the first rule that R must live in the second house and that therefore N must live in the first house. Thus, the scenario for both questions turns out to be identical:

Dec91_Game_#2_#12_diagram 2.png
The scenario above proves answer choice (A) is correct.
You do not have the required permissions to view the files attached to this post.
 JeoVela
  • Posts: 3
  • Joined: Feb 05, 2012
|
#3661
In your LSAT game type training book im having a problem with question 12 on pg 24. I have the set up right because the previous problems I had no problem with. But im confused as to why the answer is (A) is when it could also be (C) or (D)? After making the inferences from the rules and the rule in the local question I determined M would have to fill the 3rd slot followed by K in the 4th and finally O in the 5th slot in order for P >L to fit. Im confused as to answer it in a max/min fashion as well.


Thanks,

JV
User avatar
 Dave Killoran
PowerScore Staff
  • PowerScore Staff
  • Posts: 5972
  • Joined: Mar 25, 2011
|
#3663
Hey JV,

Thanks for the question. Both questions #12 and #13 in that game have identical setups (which is a very odd circumstance for two consecutive questions, and one that rarely happens). Let's walk through what occurs in the question setup in order to help answer your question.

The question stems in these two questions each establish that O must live east of the MK block. For this to occur, O must live in houses 5, 6, or 7. Consequently, O, P, and L must occupy the fifth, sixth, and seventh houses in some order, and M must then live in the third house:


... M K ( O , P > L )
1 2 3 4 5 6 7


Only N and R remain unplaced, and we can infer from the first rule that R must live in the second house and that therefore N must live in the first house. Thus, the scenario for both questions turns out to be identical:


N R M K ( O , P > L )
1 2 3 4 5 6 7


The scenario above proves answer choice (A) correct in #12, and proves answer choice (D) correct in #13.

In your question, it looks like you assumed that O had to be fifth, when it could be 5th, 6th, or 7th (as long as P > L, O can be in any position).

Please let me know if that helps. Thanks!
 medialaw111516
  • Posts: 80
  • Joined: Dec 11, 2018
|
#71084
Is diagramming the split option for M the most efficient way to get the answer for this one? I always feel like I'm doing something wrong when I just diagram both options for these types of questions.
 Rachael Wilkenfeld
PowerScore Staff
  • PowerScore Staff
  • Posts: 1419
  • Joined: Dec 15, 2011
|
#71825
Hi medialaw111516,

For this local question, we know where M has to be placed. If O is east of M, that puts L, P and O all east of M. In order for that to occur, M has to be west of K, otherwise we'd run out of room.

Let's break it down a bit.



West :longline: :longline: :longline: K :longline: :longline: :longline: East


Our rules put M immediately either before or after K. To figure out which one applies here, we have to think about how much space we have east of K. Our third rule tells us that P and L are always east of M and K. So 2 of 3 east of K slots are taken by P/L. Here, we also know that O is east of M. If M is east of K, that would put M, O, P and L all east of K, to fit in only 3 slots. That can't work. So M has to be west of K in this scenario. That's all answer choice (A) tells us.

Hope that helps!
Rachael
 medialaw111516
  • Posts: 80
  • Joined: Dec 11, 2018
|
#71900
Thanks Rachael!
 kalifaingold
  • Posts: 11
  • Joined: Jan 14, 2020
|
#73574
Administrator wrote:Complete Question Explanation
(The complete setup for this game can be found here: https://forum.powerscore.com/lsat/viewtopic.php?t=15652)

The correct answer choice is (A)

The question stems in these two questions each establish that O must live east of the MK block. For this to occur, O must live in houses 5, 6, or 7. Consequently, O, P, and L must occupy the fifth, sixth, and seventh houses in some order, and M must live in the third house:

Dec91_Game_#2_#12_diagram 1.png
Only N and R remain unplaced, and we can infer from the first rule that R must live in the second house and that therefore N must live in the first house. Thus, the scenario for both questions turns out to be identical:

Dec91_Game_#2_#12_diagram 2.png
The scenario above proves answer choice (A) is correct.
Hi, what do you mean by "Consequently, O, P, and L must occupy the fifth, sixth, and seventh houses in some order"? Wouldnt the order have to be OPL since they since we have the sequence O-P-L and there is only the 5th 6th and 7th houses left to fill?
 Jeremy Press
PowerScore Staff
  • PowerScore Staff
  • Posts: 1000
  • Joined: Jun 12, 2017
|
#73588
Hi Kali,

For this question, we actually don't have the full sequence of O - P - L. Both P and O must appear east (to the right) of M. But, once we've placed those two variables east of M, we don't know their relationship to one another. So long as O is east of M, it could appear either east or west of P. That's why we don't know the precise order of O, P, and L. There are three options, although it's probably not worth depicting each of them in a testing context. In spaces 5, 6, and 7, we could have:

1. O P L

2. P O L

3. P L O

I hope this helps!

Jeremy
 sbose
  • Posts: 19
  • Joined: May 01, 2020
|
#75332
Hi PowerScore,

I was just wondering why the answer is A when D is also true? Is it because A "must be true" as per the rules of the game, while D is only "inferred" to be true until it's proven in question 13?

Thank you so much for your help!
Srishti
 Jeremy Press
PowerScore Staff
  • PowerScore Staff
  • Posts: 1000
  • Joined: Jun 12, 2017
|
#75368
Hi Srishti!

Answer choice A is correct because it is something that is true in every single solution that is possible under the conditions of this question stem. Under the conditions of this question stem, there is no way to place the Muirs east of the Kahns, because all of the positions (5, 6, and 7) east of the Kahns must be filled by O, P, and L. Thus, however we solve the question, the Kahns will always be east of the Muirs.

Answer choice D is incorrect because it is not something that is true in every single solution that is possible under the conditions of the question stem and the game. In this question, there is a way to place the Piatts east of the Owens. That would be this solution: N R M K O P L. Since this solution is possible, answer choice D is only something that could be true, not something that always has to be true (must be true).

I hope this helps!

Jeremy

Get the most out of your LSAT Prep Plus subscription.

Analyze and track your performance with our Testing and Analytics Package.